LE4 - HTN Flashcards

1
Q

A 32-year-old G2P1 (1-0-0-1) presents for her first prenatal visit at 12 weeks of gestation. Her medical history reveals a BMI of 33 kg/m² and chronic hypertension well-controlled on medication. She denies any significant family or obstetric history.

Which of the following is the most appropriate next step in managing her pregnancy?
A. Reassure and proceed with routine prenatal care
B. Initiate low-dose aspirin at 12 weeks gestation
C. Advise strict bed rest starting at the second trimester
D. Perform biweekly fetal growth scans starting at 20 weeks

A

B. Initiate low-dose aspirin at 12 weeks gestation
Rationale: The patient has two high-risk factors for preeclampsia: chronic hypertension and obesity (BMI >30 kg/m²). Low-dose aspirin initiated between 12-28 weeks (preferably before 16 weeks) is recommended to reduce the risk of preeclampsia.

How well did you know this?
1
Not at all
2
3
4
5
Perfectly
2
Q

A 28-year-old G1 presents for her first prenatal visit at 9 weeks of gestation. She is nulliparous, has a family history of preeclampsia in her mother, and reports a low socioeconomic status.

What is the most appropriate recommendation to reduce her risk of developing preeclampsia?
A. No intervention is necessary at this time
B. Start calcium supplementation
C. Prescribe low-dose aspirin
D. Schedule early delivery at 36 weeks

A

C. Prescribe low-dose aspirin
Rationale: The patient has three moderate-risk factors: nulliparity, family history of preeclampsia, and low socioeconomic status. The presence of at least two moderate-risk factors warrants the use of low-dose aspirin as a preventive measure.

How well did you know this?
1
Not at all
2
3
4
5
Perfectly
3
Q

A 38-year-old G3P2 (2-0-0-2) presents at 10 weeks gestation for prenatal care. She has a history of preeclampsia in her second pregnancy and reports a 12-year interval since her last delivery.

What combination of factors qualifies her for preeclampsia prevention with low-dose aspirin?
A. History of preeclampsia only
B. >10-year pregnancy interval only
C. Both history of preeclampsia and >10-year pregnancy interval
D. Advanced maternal age (>35 years) and >10-year pregnancy interval

A

C. Both history of preeclampsia and >10-year pregnancy interval
Rationale: The patient has one high-risk factor (history of preeclampsia) and one moderate-risk factor (>10-year pregnancy interval). A single high-risk factor is sufficient to recommend low-dose aspirin.

How well did you know this?
1
Not at all
2
3
4
5
Perfectly
4
Q

A 29-year-old G2P1 presents for her first prenatal visit at 8 weeks gestation. She has a history of lupus, which has been stable for several years, and diabetes mellitus type 1.

Which of the following best describes her risk category for preeclampsia?
A. Moderate risk
B. High risk
C. Low risk
D. No risk

A

B. High risk
Rationale: The patient has two high-risk factors: lupus (an autoimmune disease) and type 1 diabetes mellitus. High-risk factors warrant preventive measures, such as low-dose aspirin, to lower her risk of preeclampsia.

How well did you know this?
1
Not at all
2
3
4
5
Perfectly
5
Q

A 36-year-old G1 presents at 11 weeks gestation for her first prenatal visit. She reports a family history of preeclampsia in her sister and has a history of delivering a low-birth-weight infant in a prior pregnancy.

What is the minimum number of additional moderate-risk factors required to recommend low-dose aspirin?
A. None
B. One
C. Two
D. Three

A

B. One
Rationale: The patient already has two moderate-risk factors (family history of preeclampsia and history of low birth weight). Adding just one more moderate-risk factor (e.g., nulliparity or maternal age >35 years) would warrant preeclampsia prevention with low-dose aspirin.

How well did you know this?
1
Not at all
2
3
4
5
Perfectly
6
Q

A 29-year-old G1P0 is undergoing routine prenatal screening at 12 weeks gestation. Uterine artery Doppler shows high resistance flow with a persistent diastolic notch.

What does this finding most likely indicate?
A. Normal placental development
B. Increased risk of preeclampsia due to poor trophoblastic invasion
C. Reduced fetal growth due to elevated placental growth factors
D. Early signs of preterm labor

A

B. Increased risk of preeclampsia due to poor trophoblastic invasion
Rationale: High resistance in uterine artery Doppler indices, along with a persistent diastolic notch, suggests poor endovascular trophoblast invasion, a hallmark of abnormal placentation associated with preeclampsia.

How well did you know this?
1
Not at all
2
3
4
5
Perfectly
7
Q

A 34-year-old G2P1 presents for her prenatal visit at 13 weeks gestation. The physician plans to assess preeclampsia risk using uterine artery Doppler and maternal serum biomarkers.

Which combination of predictors provides the highest sensitivity and specificity for preeclampsia risk?
A. PP13 + Uterine Artery Doppler
B. MAP + PAPP-A + Uterine Artery Doppler + PlGF
C. PlGF + VEGF + sFLT-1
D. PP13 + PAPP-A + Activin A

A

B. MAP + PAPP-A + Uterine Artery Doppler + PlGF
Rationale: The combination of MAP, PAPP-A, uterine artery Doppler, and PlGF provides a sensitivity of 93% and specificity of 95%, making it the best approach for predicting preeclampsia.

How well did you know this?
1
Not at all
2
3
4
5
Perfectly
8
Q

A 35-year-old G2P1 presents at 24 weeks gestation. Uterine artery Doppler shows abnormal flow velocity ratios and a persistent diastolic notch. Her MAP is 94 mmHg.

What does this combination of findings most likely suggest?
A. Increased risk of fetal macrosomia
B. High risk of preeclampsia with potential adverse outcomes
C. Low risk of preeclampsia and normal fetal growth
D. No significant impact on placental function

A

B. High risk of preeclampsia with potential adverse outcomes
Rationale: A persistent diastolic notch and MAP >90 mmHg are strong indicators of poor placentation and increased risk of preeclampsia, potentially leading to adverse pregnancy outcomes.

How well did you know this?
1
Not at all
2
3
4
5
Perfectly
9
Q

A 28-year-old G1P0 presents at 10 weeks gestation for her first prenatal visit. She has a BMI of 35 kg/m² and a history of chronic hypertension.

What is the most appropriate preventive measure for reducing her risk of preeclampsia?
A. Low-dose aspirin, 81 mg daily, starting at 20 weeks gestation
B. High-dose calcium, 1500-2000 mg daily, starting immediately
C. Salt restriction and bed rest starting at the second trimester
D. Vitamin C and E supplementation starting in the first trimester

A

B. High-dose calcium, 1500-2000 mg daily, starting immediately
Rationale: High-dose calcium supplementation is recommended starting as early as the first prenatal visit, especially for patients at high risk of preeclampsia. Additionally, low-dose aspirin should be initiated before 16 weeks at a higher dosage (100-150 mg daily). Salt restriction, bed rest, and vitamin supplementation are not effective preventive measures.

How well did you know this?
1
Not at all
2
3
4
5
Perfectly
10
Q

A 30-year-old G2P1 presents for her prenatal visit at 12 weeks gestation. She has a history of preeclampsia in her previous pregnancy and is currently normotensive.

What is the most appropriate preventive strategy to reduce her risk of preeclampsia?
A. Initiate low-dose aspirin, 100-150 mg daily
B. Restrict dietary salt intake
C. Recommend moderate aerobic exercise
D. Prescribe folic acid supplementation

A

A. Initiate low-dose aspirin, 100-150 mg daily
Rationale: Low-dose aspirin (100-150 mg daily) initiated before 16 weeks gestation is strongly recommended for patients with a high risk of preeclampsia, such as those with a prior history of the condition. Restricting salt, exercise, and folic acid supplementation are not effective preventive measures.

How well did you know this?
1
Not at all
2
3
4
5
Perfectly
11
Q

A 24-year-old G1P0 presents at 8 weeks gestation for her first prenatal visit. She reports a low-calcium diet. She has no significant medical history but is concerned about developing preeclampsia.

What is the most appropriate recommendation to reduce her risk?
A. Low-dose aspirin
B. High-dose calcium supplementation
C. Vitamin C and E supplementation
D. Bed rest

A

B. High-dose calcium supplementation
Rationale: For patients with inadequate dietary calcium intake, high-dose calcium supplementation (1500-2000 mg daily) is recommended starting early in pregnancy to reduce the risk of preeclampsia. Low-dose aspirin is not indicated for patients without additional high-risk factors.

How well did you know this?
1
Not at all
2
3
4
5
Perfectly
12
Q

A 35-year-old G2P1 presents at 14 weeks gestation with a history of lupus and chronic hypertension. She is currently on antihypertensive therapy.

What preventive measures should be implemented to reduce her risk of preeclampsia?
A. Low-dose aspirin and high-dose calcium supplementation
B. Exercise and salt restriction
C. Marine oil supplementation and vitamin E
D. Progestogen therapy

A

A. Low-dose aspirin and high-dose calcium supplementation
Rationale: For high-risk patients (e.g., with lupus and chronic hypertension), low-dose aspirin (100-150 mg daily) and high-dose calcium (1500-2000 mg daily) are recommended. Exercise, salt restriction, marine oil, vitamin E, and progestogens have no proven benefit in preventing preeclampsia.

How well did you know this?
1
Not at all
2
3
4
5
Perfectly
13
Q

A 27-year-old G1P0 presents at 20 weeks gestation with a BMI of 32 kg/m². She asks about preventive strategies for preeclampsia.

Which of the following strategies is NOT recommended for preeclampsia prevention?
A. High-dose calcium supplementation
B. Bed rest
C. Low-dose aspirin
D. Dietary calcium intake

A

B. Bed rest
Rationale: Bed rest is not effective in preventing preeclampsia and is therefore not recommended. High-dose calcium supplementation, low-dose aspirin, and ensuring adequate dietary calcium intake are appropriate preventive measures.

How well did you know this?
1
Not at all
2
3
4
5
Perfectly
14
Q

A 33-year-old G2P1 presents at 9 weeks gestation with a history of gestational hypertension in her previous pregnancy. She inquires about folic acid supplementation for preeclampsia prevention.

What is the role of folic acid in preeclampsia prevention?
A. Strongly recommended for all pregnancies
B. Effective only for high-risk pregnancies
C. Not recommended due to low-grade evidence
D. Recommended with low-dose aspirin

A

C. Not recommended due to low-grade evidence
Rationale: Folic acid supplementation has low-grade evidence (Level IIB) for its effectiveness in preventing preeclampsia and is not routinely recommended for this purpose.

How well did you know this?
1
Not at all
2
3
4
5
Perfectly
15
Q

A 29-year-old G1P0 with no high-risk factors asks about the timing of starting low-dose aspirin for preeclampsia prevention.

What is the most appropriate timing for initiating low-dose aspirin?
A. As soon as pregnancy is confirmed
B. Before 16 weeks gestation
C. After 20 weeks gestation
D. At the onset of preeclampsia symptoms

A

B. Before 16 weeks gestation
Rationale: Low-dose aspirin (100-150 mg daily) should be started before 16 weeks gestation and continued until 36 weeks or delivery for patients at high risk of preeclampsia. Starting after 20 weeks is less effective.

How well did you know this?
1
Not at all
2
3
4
5
Perfectly
16
Q

A 30-year-old G1P0 presents at 34 weeks gestation with blood pressure readings of 160/100 mmHg and significant proteinuria. On examination, fundoscopy reveals retinal arteriolar narrowing.

What is the primary pathological mechanism responsible for her condition?
A. Glomerular hypertrophy
B. Endothelial cell activation and vascular constriction
C. Chronic renal ischemia
D. Increased cytokine production without vascular involvement

A

B. Endothelial cell activation and vascular constriction
Rationale: The main pathology in preeclampsia is endothelial cell activation, leading to vascular constriction, increased resistance, and hypertension. Endothelial damage results in vascular dysfunction, which can be detected through retinal changes like arteriolar narrowing.

How well did you know this?
1
Not at all
2
3
4
5
Perfectly
17
Q

A 32-year-old G2P1 presents at 36 weeks gestation with severe hypertension, proteinuria, and peripheral edema. Laboratory tests reveal low glomerular filtration rate (GFR) and significant proteinuria.

What pathological process is most likely causing her renal dysfunction?
A. Ischemia-induced glomerulosclerosis
B. Glomerular endotheliosis with basement membrane thickening
C. Increased oxidative stress in the renal cortex
D. Deposition of fibrin and platelets in the renal tubules

A

B. Glomerular endotheliosis with basement membrane thickening
Rationale: The kidney is the first organ affected in preeclampsia. Glomerular endotheliosis, characterized by thickened basement membranes and increased capillary permeability, leads to decreased GFR and proteinuria.

How well did you know this?
1
Not at all
2
3
4
5
Perfectly
18
Q

A 28-year-old G1P0 at 35 weeks gestation is diagnosed with preeclampsia. Placental biopsy reveals evidence of ischemia and oxidative stress.

Which of the following best explains the systemic effects of these findings?
A. Localized placental ischemia without systemic impact
B. Oxidative stress induces cytokine release, leading to endothelial activation
C. Decreased circulating endothelial cells reduce vascular resistance
D. Ischemia primarily leads to hepatic necrosis without vascular involvement

A

B. Oxidative stress induces cytokine release, leading to endothelial activation
Rationale: Oxidative stress in the placenta provokes the release of cytokines (e.g., TNF-alpha, IL), which activate maternal endothelial cells, increasing capillary permeability and contributing to the systemic features of preeclampsia.

How well did you know this?
1
Not at all
2
3
4
5
Perfectly
19
Q

A 35-year-old G2P1 at 36 weeks gestation presents with severe preeclampsia. She has elevated liver enzymes and thrombocytopenia.

What is the underlying mechanism of these complications?
A. Ischemia and necrosis due to diminished organ perfusion
B. Platelet aggregation in hepatic veins
C. Increased glomerular filtration rate
D. Increased production of placental angiogenic factors

A

A. Ischemia and necrosis due to diminished organ perfusion
Rationale: In preeclampsia, systemic ischemia leads to end-organ damage, including hepatic necrosis and hemorrhage, manifesting as elevated liver enzymes and thrombocytopenia.

How well did you know this?
1
Not at all
2
3
4
5
Perfectly
20
Q

A 29-year-old G1P0 presents at 33 weeks gestation with hypertension, proteinuria, and visual disturbances. Examination reveals subconjunctival hemorrhages and arteriolar narrowing.

What explains the visual symptoms in this patient?
A. Elevated intracranial pressure due to eclampsia
B. Oxidative stress affecting retinal perfusion
C. Vasospasm causing retinal ischemia and arteriolar damage
D. Increased permeability of retinal vessels due to proteinuria

A

C. Vasospasm causing retinal ischemia and arteriolar damage
Rationale: Visual symptoms in preeclampsia are often due to vasospasm and ischemia affecting the retinal arterioles, resulting in arteriolar narrowing, hemorrhages, and potential visual disturbances.

How well did you know this?
1
Not at all
2
3
4
5
Perfectly
21
Q

A 37-year-old G3P2 at 34 weeks gestation presents with worsening hypertension and severe proteinuria. Laboratory studies show elevated TNF-alpha levels and circulating endothelial microparticles (EMP).

What is the significance of these findings in the context of preeclampsia?
A. TNF-alpha and EMP indicate renal-specific damage
B. EMP levels predict immediate onset of eclampsia
C. Cytokine release and EMP reflect systemic endothelial activation
D. TNF-alpha directly reduces GFR without vascular involvement

A

C. Cytokine release and EMP reflect systemic endothelial activation
Rationale: Increased levels of TNF-alpha and endothelial microparticles in preeclampsia are markers of systemic endothelial activation, which contributes to vascular dysfunction and the clinical manifestations of the disease.

How well did you know this?
1
Not at all
2
3
4
5
Perfectly
22
Q

A 34-year-old G2P1 with preeclampsia develops hemolysis, elevated liver enzymes, and low platelets (HELLP syndrome).

What is the primary pathology underlying these findings?
A. Hepatic microvascular thrombosis and necrosis
B. Placental ischemia without systemic involvement
C. Glomerular hyperperfusion and protein loss
D. Cytokine-mediated vascular constriction in the kidneys

A

A. Hepatic microvascular thrombosis and necrosis
Rationale: HELLP syndrome results from severe systemic endothelial damage leading to microvascular thrombosis, ischemia, and necrosis, especially in the liver, causing elevated liver enzymes and hemolysis.

How well did you know this?
1
Not at all
2
3
4
5
Perfectly
23
Q

A 29-year-old G1P0 presents at 32 weeks gestation with blood pressure readings of 150/95 mmHg on two separate occasions 6 hours apart. She reports no headaches, visual changes, or right upper quadrant pain. Urinalysis shows 2+ protein.

What is the most likely diagnosis?
A. Gestational hypertension
B. Chronic hypertension
C. Preeclampsia without severe features
D. Preeclampsia with severe features

What is the most appropriate management for this patient?
A. Immediate delivery
B. Start magnesium sulfate prophylaxis
C. Expectant management with close monitoring
D. Initiate antihypertensive therapy

A

C. Preeclampsia without severe features
Rationale: Preeclampsia is diagnosed based on hypertension (≥140/90 mmHg after 20 weeks gestation) and proteinuria (≥1+ on dipstick). The absence of severe features such as severe hypertension, end-organ dysfunction, or symptoms indicates preeclampsia without severe features.

C. Expectant management with close monitoring
Rationale: Preeclampsia without severe features can be managed expectantly, particularly if gestational age is <37 weeks. Regular monitoring of BP, proteinuria, maternal symptoms, and fetal well-being is essential.

How well did you know this?
1
Not at all
2
3
4
5
Perfectly
24
Q

A 35-year-old G2P1 at 34 weeks gestation presents with severe hypertension (170/110 mmHg), proteinuria, and persistent epigastric pain. Lab results reveal AST and ALT levels 3x normal and platelets of 90,000/mm³.

What is the most likely diagnosis?
A. Preeclampsia without severe features
B. Preeclampsia with severe features
C. Eclampsia
D. HELLP syndrome

What is the most appropriate management for this patient?
A. Continue pregnancy with antihypertensive therapy
B. Immediate delivery
C. Administer magnesium sulfate and delay delivery until 37 weeks
D. Observe for progression to eclampsia

A

B. Preeclampsia with severe features
Rationale: Severe hypertension, elevated liver enzymes, and thrombocytopenia (<100,000/mm³) classify this case as preeclampsia with severe features. Epigastric pain suggests hepatic involvement, a common severe feature.

B. Immediate delivery
Rationale: Delivery is the definitive treatment for preeclampsia with severe features at or beyond 34 weeks gestation. Magnesium sulfate should be administered for seizure prophylaxis, and antihypertensive therapy can be initiated to control BP.

How well did you know this?
1
Not at all
2
3
4
5
Perfectly
25
Q

A 27-year-old G1P0 presents at 31 weeks gestation with generalized tonic-clonic seizures. BP is 160/105 mmHg, and urinalysis shows 3+ protein.

What is the most likely diagnosis?
A. Eclampsia
B. Severe preeclampsia
C. Chronic hypertension with superimposed preeclampsia
D. Epilepsy

What is the first-line treatment to control the seizures?
A. Diazepam
B. Magnesium sulfate
C. Phenytoin
D. Labetalol

A

A. Eclampsia
Rationale: Eclampsia is characterized by generalized tonic-clonic seizures in a patient with preeclampsia and no other identifiable causes.

B. Magnesium sulfate
Rationale: Magnesium sulfate is the drug of choice for both prevention and treatment of seizures in eclampsia. Diazepam and phenytoin are not first-line treatments for eclampsia-related seizures.

How well did you know this?
1
Not at all
2
3
4
5
Perfectly
26
Q

A 30-year-old G2P1 presents at 36 weeks gestation with BP of 165/110 mmHg. She complains of severe headaches and visual disturbances. Lab results show normal platelets and liver enzymes but a serum creatinine of 1.4 mg/dL.

What is the most appropriate classification of her condition?
A. Gestational hypertension
B. Preeclampsia without severe features
C. Preeclampsia with severe features
D. Chronic hypertension

What is the next best step in management?
A. Initiate labetalol and expectant management
B. Immediate delivery after stabilizing BP and administering magnesium sulfate
C. Observe and repeat BP readings in 4 hours
D. Administer aspirin and repeat labs in 1 week

A

C. Preeclampsia with severe features
Rationale: Severe hypertension, new-onset cerebral symptoms (headache, visual changes), and renal insufficiency (creatinine >1.1 mg/dL) meet the criteria for preeclampsia with severe features.

B. Immediate delivery after stabilizing BP and administering magnesium sulfate
Rationale: For preeclampsia with severe features at ≥34 weeks, delivery is the definitive treatment. Magnesium sulfate prevents seizures, and antihypertensive therapy prevents complications such as stroke.

How well did you know this?
1
Not at all
2
3
4
5
Perfectly
27
Q

A 34-year-old G3P2 is admitted postpartum after a spontaneous vaginal delivery 2 days ago. She develops generalized tonic-clonic seizures. BP is 150/95 mmHg, and urinalysis shows proteinuria.

What is the most likely cause of her seizures?
A. Stroke
B. Postpartum eclampsia
C. Epilepsy
D. Hypoglycemia

What is the most appropriate management for this patient?
A. Administer magnesium sulfate and continue monitoring
B. Initiate phenytoin therapy for seizure control
C. Start antihypertensive therapy only
D. Perform brain imaging immediately

A

B. Postpartum eclampsia
Rationale: Eclampsia can occur postpartum, typically within 48 hours but sometimes later. The presence of hypertension and proteinuria supports the diagnosis.

A. Administer magnesium sulfate and continue monitoring
Rationale: Magnesium sulfate is the first-line treatment for eclampsia, including postpartum cases. Brain imaging is reserved for atypical presentations or if the patient does not improve.

How well did you know this?
1
Not at all
2
3
4
5
Perfectly
28
Q

A 34-year-old G2P1 presents at 18 weeks gestation with a BP of 145/95 mmHg. She denies any previous history of hypertension but notes that her BP was borderline elevated before conception. Laboratory tests show no proteinuria or systemic findings.

What is the most likely diagnosis?
A. Preeclampsia without severe features
B. Chronic hypertension
C. Gestational hypertension
D. Chronic hypertension with superimposed preeclampsia

Follow-Up Question
What is the most appropriate management for this patient?
A. Initiate magnesium sulfate prophylaxis
B. Monitor BP closely and repeat testing in 4 weeks
C. Prescribe low-dose aspirin and initiate antihypertensive therapy if BP exceeds 160/105 mmHg
D. Immediate delivery due to maternal risk

A

Answer: B. Chronic hypertension
Rationale: Hypertension diagnosed before 20 weeks gestation is classified as chronic hypertension, even in the absence of proteinuria or systemic findings.

Answer: C. Prescribe low-dose aspirin and initiate antihypertensive therapy if BP exceeds 160/105 mmHg
Rationale: Chronic hypertension requires close monitoring and preventive measures, such as low-dose aspirin to reduce preeclampsia risk. Antihypertensives are initiated for severe hypertension (≥160/105 mmHg).

How well did you know this?
1
Not at all
2
3
4
5
Perfectly
29
Q

A 36-year-old G3P2 at 28 weeks gestation has a history of chronic hypertension. Her BP today is 180/110 mmHg. Laboratory results show proteinuria (2+ on dipstick), platelets of 90,000/mm³, and AST elevated to 80 U/L.

What is the most likely diagnosis?
A. Chronic hypertension
B. Chronic hypertension with superimposed preeclampsia
C. Gestational hypertension
D. Preeclampsia with severe features

Follow-Up Question
What is the next best step in management?
A. Immediate delivery after stabilizing BP and administering magnesium sulfate
B. Continue monitoring and delay delivery until 37 weeks
C. Prescribe aspirin and calcium supplementation
D. Perform a repeat BP measurement in 4 hours

A

Answer: B. Chronic hypertension with superimposed preeclampsia
Rationale: Chronic hypertension complicated by new-onset proteinuria, thrombocytopenia, and elevated liver enzymes meets the criteria for chronic hypertension with superimposed preeclampsia.

Answer: A. Immediate delivery after stabilizing BP and administering magnesium sulfate
Rationale: Chronic hypertension with superimposed preeclampsia and severe features requires immediate delivery if gestational age is ≥34 weeks or maternal-fetal condition worsens. Magnesium sulfate is administered for seizure prophylaxis.

How well did you know this?
1
Not at all
2
3
4
5
Perfectly
30
Q

A 28-year-old G1P0 at 32 weeks gestation presents with a BP of 150/95 mmHg on two separate occasions 6 hours apart. Urinalysis is negative for proteinuria, and no systemic symptoms are reported.

What is the most likely diagnosis?
A. Chronic hypertension
B. Chronic hypertension with superimposed preeclampsia
C. Gestational hypertension
D. Preeclampsia

Follow-Up Question
What is the most appropriate management for this patient?
A. Immediate delivery
B. Continue monitoring BP and fetal well-being
C. Initiate antihypertensive therapy
D. Administer magnesium sulfate prophylaxis

Answer: B. Continue monitoring BP and fetal well-being
Rationale: Gestational hypertension without severe features is managed expectantly with regular monitoring of BP and fetal well-being. Antihypertensives are only indicated for severe hypertension (≥160/105 mmHg).

A

Answer: C. Gestational hypertension
Rationale: Hypertension diagnosed after 20 weeks gestation in a previously normotensive woman, without proteinuria or systemic features, is classified as gestational hypertension.

Answer: B. Continue monitoring BP and fetal well-being
Rationale: Gestational hypertension without severe features is managed expectantly with regular monitoring of BP and fetal well-being. Antihypertensives are only indicated for severe hypertension (≥160/105 mmHg).

How well did you know this?
1
Not at all
2
3
4
5
Perfectly
31
Q

A 30-year-old G2P1 at 24 weeks gestation presents with a BP of 165/105 mmHg. Laboratory results reveal normal platelets, creatinine, and liver enzymes. Urinalysis shows no proteinuria.

What is the most likely diagnosis?
A. Severe gestational hypertension
B. Preeclampsia without severe features
C. Chronic hypertension
D. Chronic hypertension with superimposed preeclampsia

Follow-Up Question
What is the next best step in management?
A. Immediate delivery
B. Initiate antihypertensive therapy and close monitoring
C. Administer magnesium sulfate
D. Repeat BP measurements in 1 week

A

Answer: A. Severe gestational hypertension
Rationale: Hypertension ≥160/110 mmHg without proteinuria or systemic findings is classified as severe gestational hypertension, which requires close monitoring to differentiate from preeclampsia.

Answer: B. Initiate antihypertensive therapy and close monitoring
Rationale: Severe hypertension (≥160/110 mmHg) requires antihypertensive therapy to reduce maternal risks, such as stroke, even in the absence of preeclampsia. Delivery is not indicated unless maternal or fetal conditions worsen.

How well did you know this?
1
Not at all
2
3
4
5
Perfectly
32
Q

A 32-year-old G2P1 presents at 36 weeks gestation with complaints of fatigue, nausea, and right upper quadrant pain. Blood pressure is 140/90 mmHg. Laboratory results show:

Platelets: 80,000/mm³
ALT: 120 U/L
LDH: 700 U/L
Peripheral smear: Crenated red cells and reticulocytosis
What is the most likely diagnosis?
A. Preeclampsia with severe features
B. HELLP syndrome
C. Acute fatty liver of pregnancy (AFLP)
D. Viral hepatitis

Follow-Up Question
What is the most appropriate next step in management?
A. Administer plasma exchange therapy
B. Start betamethasone and delay delivery for 24-48 hours
C. Immediate delivery after stabilizing maternal condition
D. Monitor and repeat laboratory studies in 24 hours

A

Answer: B. HELLP syndrome
Rationale: The patient meets the laboratory criteria for HELLP syndrome, including thrombocytopenia (<100,000/mm³), elevated liver enzymes (ALT >70 U/L, LDH >600 U/L), and hemolysis (abnormal peripheral smear and reticulocytosis). The presence of right upper quadrant pain and mild hypertension supports this diagnosis.

Answer: C. Immediate delivery after stabilizing maternal condition
Rationale: At 36 weeks gestation, delivery is the cornerstone of therapy for HELLP syndrome. Maternal stabilization with antihypertensives, magnesium sulfate, and platelet transfusion (if necessary) should precede delivery.

How well did you know this?
1
Not at all
2
3
4
5
Perfectly
33
Q

A 28-year-old G1P0 at 32 weeks gestation presents with complaints of malaise and epigastric pain. Blood pressure is 125/85 mmHg, and there is 1+ proteinuria. Laboratory findings include:

Platelets: 90,000/mm³
AST: 95 U/L
LDH: 650 U/L
Which of the following best differentiates HELLP syndrome from preeclampsia in this patient?
A. Proteinuria
B. Severe hypertension
C. Hemolysis on peripheral smear
D. Elevated transaminases

Follow-Up Question
What is the most appropriate management if the fetus is 32 weeks gestation and stable?
A. Immediate delivery without further intervention
B. Administer betamethasone and delay delivery for 24-48 hours
C. Start plasma exchange therapy
D. Observe and monitor for progression of symptoms

A

Answer: C. Hemolysis on peripheral smear
Rationale: Hemolysis is a hallmark of HELLP syndrome that differentiates it from preeclampsia. Elevated liver enzymes and thrombocytopenia are shared features, but hemolysis is unique to HELLP syndrome.

Answer: B. Administer betamethasone and delay delivery for 24-48 hours
Rationale: At 32 weeks, delivery can be delayed by 24-48 hours if maternal and fetal conditions are stable. Betamethasone is administered to promote fetal lung maturity.

How well did you know this?
1
Not at all
2
3
4
5
Perfectly
34
Q

A 35-year-old G3P2 presents postpartum 48 hours after cesarean delivery with complaints of malaise, nausea, and right upper quadrant pain. Laboratory tests show:

Platelets: 70,000/mm³
ALT: 105 U/L
LDH: 800 U/L
What is the most likely diagnosis?
A. Postpartum preeclampsia
B. Acute fatty liver of pregnancy (AFLP)
C. HELLP syndrome
D. Idiopathic thrombocytopenic purpura (ITP)

Follow-Up Question
What supportive measures are recommended for this patient?
A. Dexamethasone for platelet stabilization
B. Platelet transfusion to maintain platelets ≥40,000 cells/μL
C. Plasma exchange therapy to remove hemolytic factors
D. Observation and symptomatic management

A

Answer: C. HELLP syndrome
Rationale: HELLP syndrome can occur postpartum, typically within 48 hours of delivery. Laboratory findings of thrombocytopenia, elevated liver enzymes, and hemolysis (LDH >600 U/L) confirm the diagnosis.

Answer: B. Platelet transfusion to maintain platelets ≥40,000 cells/μL
Rationale: Platelet transfusion is indicated in HELLP syndrome to stabilize platelet counts, particularly if invasive procedures are necessary. Dexamethasone and plasma exchange are not recommended for HELLP syndrome.

How well did you know this?
1
Not at all
2
3
4
5
Perfectly
35
Q

A 30-year-old G2P1 at 28 weeks gestation presents with nausea, fatigue, and mild hypertension (BP 140/90 mmHg). Lab findings include:

Platelets: 95,000/mm³
AST: 100 U/L
LDH: 750 U/L
Which of the following conditions should be included in the differential diagnosis?
A. Viral hepatitis
B. Acute fatty liver of pregnancy (AFLP)
C. Thrombotic thrombocytopenic purpura (TTP)
D. All of the above

Follow-Up Question
What laboratory test can help differentiate HELLP syndrome from AFLP?
A. ALT and AST levels
B. Blood glucose levels
C. Peripheral smear findings
D. Proteinuria measurement

A

Answer: D. All of the above
Rationale: HELLP syndrome shares overlapping features with other conditions, such as viral hepatitis, AFLP, and TTP. A thorough history, physical exam, and additional testing are necessary to confirm the diagnosis.

Answer: B. Blood glucose levels
Rationale: AFLP often presents with hypoglycemia, which is not a feature of HELLP syndrome. Blood glucose levels can help differentiate these conditions.

How well did you know this?
1
Not at all
2
3
4
5
Perfectly
36
Q

A 31-year-old G2P1 presents at 34 weeks gestation with complaints of malaise and right upper quadrant tenderness. Blood pressure is 145/90 mmHg, and urinalysis shows 2+ protein. Laboratory results are as follows:

Platelets: 85,000/mm³
AST: 90 U/L
LDH: 700 U/L
What is the most appropriate classification of this condition based on the Tennessee criteria?
A. Complete HELLP syndrome
B. Partial HELLP syndrome (ELLP)
C. Partial HELLP syndrome (EL)
D. Severe preeclampsia

Follow-Up Question
What is the most appropriate management for this patient?
A. Observe and repeat laboratory tests in 24 hours
B. Immediate delivery after stabilizing maternal condition
C. Administer betamethasone and delay delivery for 48 hours
D. Start dexamethasone for platelet stabilization

A

Answer: A. Complete HELLP syndrome
Rationale: The patient meets the criteria for complete HELLP syndrome with thrombocytopenia (<100,000/mm³), elevated AST (>70 U/L), and elevated LDH (>600 U/L).

Answer: B. Immediate delivery after stabilizing maternal condition
Rationale: At 34 weeks gestation, delivery is the definitive treatment for complete HELLP syndrome. Maternal stabilization with magnesium sulfate for seizure prophylaxis and antihypertensive therapy is essential before delivery.

How well did you know this?
1
Not at all
2
3
4
5
Perfectly
37
Q

A 28-year-old G1P0 at 32 weeks gestation presents with proteinuria, malaise, and right upper quadrant tenderness. Blood pressure is 135/85 mmHg. Laboratory findings include:

Platelets: 120,000/mm³
AST: 100 U/L
LDH: 700 U/L
What is the most appropriate classification based on the Tennessee criteria?
A. Complete HELLP syndrome
B. Partial HELLP syndrome (EL)
C. Partial HELLP syndrome (ELLP)
D. Severe preeclampsia

Follow-Up Question
What is the next step in management if maternal and fetal conditions are stable?
A. Delay delivery for 24-48 hours with betamethasone administration
B. Immediate delivery regardless of stability
C. Start dexamethasone for liver enzyme control
D. Perform plasma exchange therapy

A

Answer: B. Partial HELLP syndrome (EL)
Rationale: The patient meets the criteria for partial HELLP syndrome with elevated liver enzymes (AST >70 U/L) and LDH >600 U/L but does not meet the platelet threshold (<100,000/mm³) for complete HELLP syndrome.

Answer: A. Delay delivery for 24-48 hours with betamethasone administration
Rationale: If maternal and fetal conditions are stable, delivery can be delayed to allow for betamethasone administration, which promotes fetal lung maturity. This is particularly important at 32 weeks gestation

How well did you know this?
1
Not at all
2
3
4
5
Perfectly
38
Q

A 35-year-old G2P1 presents postpartum with complaints of fatigue and right upper quadrant pain. Laboratory tests reveal the following:

Platelets: 95,000/mm³
AST: 85 U/L
LDH: 650 U/L
Peripheral smear: Hemolysis present
What is the Tennessee classification for this condition?
A. Complete HELLP syndrome
B. Partial HELLP syndrome (HEL)
C. Partial HELLP syndrome (ELLP)
D. Severe preeclampsia

Follow-Up Question
What supportive therapy is recommended in this patient?
A. Platelet transfusion
B. Plasma exchange therapy
C. Magnesium sulfate and close monitoring
D. Immediate hysterectomy

A

Answer: B. Partial HELLP syndrome (HEL)
Rationale: The patient demonstrates hemolysis, elevated liver enzymes, and low platelets (though >100,000/mm³). This fits the criteria for partial HELLP syndrome (HEL).

Answer: C. Magnesium sulfate and close monitoring
Rationale: Postpartum HELLP syndrome requires supportive care, including seizure prophylaxis with magnesium sulfate and close monitoring of maternal condition. Platelet transfusion is not indicated unless platelet count is <40,000/mm³ or delivery is planned.

How well did you know this?
1
Not at all
2
3
4
5
Perfectly
39
Q

Question 4
A 30-year-old G3P2 at 36 weeks gestation presents with hypertension, proteinuria, malaise, and right upper quadrant pain. Laboratory findings include:

Platelets: 80,000/mm³
AST: 110 U/L
LDH: 900 U/L
What percentage of patients with HELLP syndrome typically present with proteinuria?
A. 50%
B. 65%
C. 86-100%
D. 90-95%

Follow-Up Question
What is the definitive management for this patient?
A. Observation with repeat labs in 12 hours
B. Immediate delivery after maternal stabilization
C. Administer antihypertensives and observe until 37 weeks
D. Administer plasma exchange therapy

A

Answer: C. 86-100%
Rationale: Proteinuria is a common feature of HELLP syndrome and is present in 86-100% of cases, distinguishing it from some conditions in the differential diagnosis.

Answer: B. Immediate delivery after maternal stabilization
Rationale: At 36 weeks gestation, delivery is the definitive treatment for HELLP syndrome. Stabilization of maternal condition with antihypertensives and magnesium sulfate should precede delivery.

How well did you know this?
1
Not at all
2
3
4
5
Perfectly
40
Q

Question 1
A 30-year-old G1P0 presents at 33 weeks gestation with mild hypertension (BP 145/90 mmHg) and 1+ proteinuria. She denies any symptoms such as headache, visual changes, or right upper quadrant pain. Fetal growth and amniotic fluid index are within normal limits.

What is the most appropriate management for this patient?
A. Immediate delivery
B. Admit for inpatient monitoring and initiate antihypertensive therapy
C. Home management with close maternal and fetal monitoring
D. Administer antenatal corticosteroids and plan delivery at 34 weeks

Follow-Up Question
What instructions should be provided for home monitoring in this patient?
A. Blood pressure should be measured weekly
B. Perform daily fetal kick counts and report any decreased movements
C. Non-stress testing should be performed daily
D. Avoid all physical activity until delivery

A

Answer: C. Home management with close maternal and fetal monitoring
Rationale: This patient has preeclampsia without severe features at <34 weeks gestation, making her eligible for home management. Regular BP monitoring, weekly lab tests, and antenatal testing (e.g., NST) are required.

Answer: B. Perform daily fetal kick counts and report any decreased movements
Rationale: Patients on home management should monitor BP at least twice daily, perform daily fetal kick counts, and promptly report symptoms such as severe headache, visual changes, or decreased fetal movements.

How well did you know this?
1
Not at all
2
3
4
5
Perfectly
41
Q

A 28-year-old G2P1 at 35 weeks gestation presents with severe hypertension (BP 170/110 mmHg) and proteinuria. She reports a persistent headache and visual disturbances. Laboratory results show platelet count 90,000/mm³ and AST 100 U/L.

What is the next best step in management?
A. Initiate magnesium sulfate and plan for immediate delivery
B. Admit for observation and repeat labs in 24 hours
C. Home management with close BP monitoring
D. Delay delivery and administer antenatal corticosteroids

A

Answer: A. Initiate magnesium sulfate and plan for immediate delivery
Rationale: The patient has preeclampsia with severe features, including severe hypertension, headache, visual changes, thrombocytopenia, and elevated liver enzymes. Immediate delivery is indicated after maternal stabilization with magnesium sulfate and antihypertensives.

How well did you know this?
1
Not at all
2
3
4
5
Perfectly
42
Q

Question 3
A 34-year-old G1P0 presents at 36 weeks gestation with BP 150/95 mmHg and 2+ proteinuria. She denies symptoms of severe preeclampsia, and fetal well-being is reassuring on NST.

What is the recommended management for this patient?
A. Immediate delivery
B. Expectant management until 37 weeks
C. Initiate magnesium sulfate and delay delivery
D. Start antihypertensives and plan for delivery at 39 weeks

Follow-Up Question
What additional steps should be taken to manage this patient prior to delivery?
A. Administer antenatal corticosteroids for fetal lung maturity
B. Monitor for seizure prophylaxis with magnesium sulfate
C. Delay delivery and repeat NST in 48 hours
D. Perform an urgent cesarean section

A

Answer: A. Immediate delivery
Rationale: At 36 weeks, delivery is the preferred management for preeclampsia, even in the absence of severe features, as fetal lungs are mature and the risks of continuing pregnancy outweigh the benefits.

Answer: B. Monitor for seizure prophylaxis with magnesium sulfate
Rationale: Seizure prophylaxis with magnesium sulfate is indicated for patients with preeclampsia to prevent progression to eclampsia. Delivery timing is not dependent on NST results if preeclampsia is confirmed.

How well did you know this?
1
Not at all
2
3
4
5
Perfectly
43
Q

A 29-year-old G2P1 at 32 weeks gestation is being managed at home for gestational hypertension. Her latest BP readings are stable at 135/85 mmHg, but she reports a persistent headache and decreased fetal movements.

What is the most appropriate next step?
A. Continue home monitoring and schedule an antenatal visit in 1 week
B. Admit to the hospital for evaluation and monitoring
C. Perform an urgent growth ultrasound and NST at the clinic
D. Administer antihypertensives and continue home management

A

B. Admit to the hospital for evaluation and monitoring
Rationale: New symptoms such as headache and decreased fetal movements warrant hospital admission for further evaluation and monitoring, as they may indicate progression to preeclampsia with severe features.

How well did you know this?
1
Not at all
2
3
4
5
Perfectly
44
Q

A 32-year-old G1P0 with mild preeclampsia is being managed expectantly at 33 weeks gestation. Fetal NST and Doppler studies are normal, and BP is stable.

When should delivery be planned for this patient?
A. At 34 weeks gestation
B. At 37 weeks gestation
C. When fetal growth restriction or oligohydramnios is detected
D. Only if severe features develop

A

A. At 34 weeks gestation
Rationale: Delivery is recommended at 34 weeks gestation for preeclampsia, even without severe features, as fetal lung maturity is expected and maternal/fetal risks increase with prolonged pregnancy.

How well did you know this?
1
Not at all
2
3
4
5
Perfectly
45
Q

A 29-year-old G1P0 presents at 35 weeks gestation with BP 170/110 mmHg on two separate readings 4 hours apart. She reports severe headaches and blurry vision. Laboratory results show platelet count 90,000/mm³, AST 95 U/L, and 2+ proteinuria.

What is the most appropriate next step in management?
A. Admit the patient for stabilization and immediate delivery
B. Initiate antihypertensives and discharge for home monitoring
C. Delay delivery and administer antenatal corticosteroids
D. Observe and repeat laboratory tests in 48 hours

Follow-Up Question
Which interventions should be included in the patient’s management plan before delivery?
A. Administer magnesium sulfate for seizure prophylaxis and initiate antihypertensives
B. Perform emergency cesarean delivery without further stabilization
C. Administer plasma exchange therapy
D. Monitor BP and observe without medications

A

Answer: A. Admit the patient for stabilization and immediate delivery
Rationale: The patient has severe preeclampsia with features such as severe hypertension, headache, visual disturbances, thrombocytopenia, and elevated liver enzymes. Immediate delivery is indicated for maternal and fetal safety as the gestational age is >34 weeks.

Answer: A. Administer magnesium sulfate for seizure prophylaxis and initiate antihypertensives
Rationale: Stabilizing maternal condition with magnesium sulfate and antihypertensives is essential to reduce the risk of seizures and prevent hypertensive complications like stroke before delivery.

46
Q

A 32-year-old G2P1 at 33 weeks gestation is admitted with BP 160/105 mmHg, 2+ proteinuria, and right upper quadrant pain. Fetal ultrasound reveals growth restriction and oligohydramnios.

What is the most appropriate next step in management?
A. Immediate delivery after maternal stabilization
B. Expectant management with frequent maternal and fetal monitoring
C. Administer corticosteroids and plan delivery at 34 weeks
D. Discharge the patient with instructions for home BP monitoring

Follow-Up Question
Which fetal surveillance methods are recommended during expectant management?
A. Weekly NSTs and monthly growth ultrasounds
B. Daily NSTs and biweekly Doppler studies
C. Biophysical profile (BPP) only at admission
D. No additional fetal surveillance if NST is initially normal

A

Answer: B. Expectant management with frequent maternal and fetal monitoring
Rationale: At <34 weeks gestation, expectant management may be acceptable if maternal and fetal conditions are stable. Close monitoring, including NSTs, Doppler studies, and maternal labs, is critical. Delivery is indicated if deterioration occurs.

Answer: B. Daily NSTs and biweekly Doppler studies
Rationale: Daily NSTs and frequent Doppler studies are essential for monitoring fetal well-being during expectant management in severe preeclampsia, especially when fetal growth restriction and oligohydramnios are present.

47
Q

A 34-year-old G3P2 presents at 31 weeks gestation with BP 155/95 mmHg and 1+ proteinuria. She denies any symptoms of severe preeclampsia. Fetal Doppler studies show abnormal umbilical artery velocimetry.

What is the most likely fetal complication in this patient?
A. Fetal macrosomia
B. Oligohydramnios
C. Fetal growth restriction (FGR)
D. Preterm labor

Follow-Up Question
How should this patient be managed if maternal and fetal conditions remain stable?
A. Admit for continuous monitoring until delivery at 34 weeks
B. Continue outpatient management with weekly antenatal testing
C. Delay delivery and perform daily Doppler studies
D. Immediate delivery due to abnormal Doppler findings

A

Answer: C. Fetal growth restriction (FGR)
Rationale: Fetal growth restriction is the most common fetal morbidity in preeclampsia due to uteroplacental insufficiency, often detected through abnormal Doppler velocimetry.

Answer: A. Admit for continuous monitoring until delivery at 34 weeks
Rationale: Abnormal Doppler findings in preeclampsia warrant inpatient management with close maternal and fetal monitoring. Delivery is typically recommended at 34 weeks to minimize risks.

48
Q

A 30-year-old G2P1 presents at 36 weeks gestation with BP 165/110 mmHg, 3+ proteinuria, and severe right upper quadrant pain. Fetal NST shows late decelerations, and biophysical profile (BPP) is 4/10.

What is the most appropriate management for this patient?
A. Expectant management with daily NSTs and BPPs
B. Immediate delivery via cesarean section
C. Administer antihypertensives and observe for 48 hours
D. Perform emergency Doppler studies to confirm fetal status

A

B. Immediate delivery via cesarean section
Rationale: Severe preeclampsia with abnormal fetal surveillance (late decelerations, low BPP) requires immediate delivery to prevent further maternal and fetal compromise. Cesarean delivery is indicated due to non-reassuring fetal status.

49
Q

A 28-year-old G1P0 at 30 weeks gestation is admitted with severe preeclampsia. BP is 160/105 mmHg despite antihypertensive therapy, and platelet count is 95,000/mm³. Fetal NST is normal.

What is the most important consideration for delivery timing?
A. Delivery should occur only after fetal lung maturity is confirmed
B. Delivery is recommended immediately if maternal condition deteriorates
C. Delivery should be delayed until 37 weeks if fetal status remains reassuring
D. Delivery timing is based solely on BP control

A

B. Delivery is recommended immediately if maternal condition deteriorates
Rationale: Severe preeclampsia at <34 weeks may warrant expectant management if maternal and fetal conditions are stable. Immediate delivery is required if there is any maternal or fetal deterioration.

50
Q

Question 1
A 32-year-old G1P0 presents at 34 weeks gestation with a BP of 165/110 mmHg on two separate readings 4 hours apart. She reports no visual changes or headache. Fetal heart rate tracing is reassuring.

What is the goal of antihypertensive therapy in this patient?
A. Normalize BP to <120/80 mmHg
B. Maintain BP in the range of 140-150/90-100 mmHg
C. Gradually reduce BP to <130/85 mmHg over 48 hours
D. Prevent the progression of preeclampsia

Follow-Up Question
What is the most appropriate first-line antihypertensive for this patient?
A. Sublingual nifedipine
B. Hydralazine IV
C. Oral labetalol
D. Magnesium sulfate

A

Answer: B. Maintain BP in the range of 140-150/90-100 mmHg
Rationale: The goal of antihypertensive therapy in preeclampsia is not to normalize BP but to achieve a safe target range of 140-150/90-100 mmHg to reduce the risk of maternal complications without compromising uteroplacental blood flow.

Answer: C. Oral labetalol
Rationale: Oral labetalol is a preferred first-line antihypertensive due to its efficacy and safety profile. Sublingual nifedipine is avoided as it can cause a rapid BP drop and compromise uteroplacental blood flow. Magnesium sulfate is used for seizure prophylaxis, not BP management.

51
Q

Question 2
A 36-year-old G2P1 at 32 weeks gestation is admitted with BP 170/115 mmHg, severe headache, and blurred vision. She is started on IV hydralazine for acute BP control.

Which of the following complications should be closely monitored during hydralazine therapy?
A. Fetal hypoxia due to uteroplacental insufficiency
B. Maternal bradycardia
C. Sudden worsening of proteinuria
D. Pulmonary edema

Follow-Up Question
What is the next best step if maternal BP remains uncontrolled despite hydralazine therapy?
A. Add sublingual nifedipine
B. Administer magnesium sulfate
C. Switch to IV labetalol
D. Perform immediate cesarean delivery

A

Answer: A. Fetal hypoxia due to uteroplacental insufficiency
Rationale: Rapid BP reduction with hydralazine can cause maternal hypotension and reduced uteroplacental blood flow, leading to fetal hypoxia. Gradual dose adjustments are critical to avoid such complications.

Answer: C. Switch to IV labetalol
Rationale: IV labetalol is an effective alternative for acute BP control in cases where hydralazine is insufficient. Sublingual nifedipine is contraindicated due to the risk of rapid BP drops.

52
Q

A 28-year-old G1P0 presents at 33 weeks gestation with BP 145/95 mmHg and mild proteinuria. Fetal well-being is reassuring, and there are no signs of severe features.

What is the most appropriate management for this patient?
A. Start oral nifedipine to lower BP
B. Monitor BP without initiating antihypertensives
C. Administer IV labetalol to prevent progression
D. Plan for immediate delivery

Follow-Up Question
What is the primary reason for avoiding antihypertensive therapy in this case?
A. To prevent hypotension and decreased uteroplacental blood flow
B. To avoid masking progression to severe preeclampsia
C. Because antihypertensives do not improve fetal outcomes
D. Because antihypertensives increase the risk of placental abruption

A

Answer: B. Monitor BP without initiating antihypertensives
Rationale: Antihypertensive therapy is not indicated for mild/moderate hypertension (BP <160/110 mmHg) in the absence of severe features. Close monitoring is sufficient in such cases.

Answer: A. To prevent hypotension and decreased uteroplacental blood flow
Rationale: Excessive lowering of BP can compromise uteroplacental blood flow, leading to fetal hypoxia or distress. Therapy is reserved for severe hypertension.

53
Q

A 35-year-old G3P2 with preeclampsia at 34 weeks gestation has BP of 175/115 mmHg. She is started on IV labetalol, and her BP reduces to 135/85 mmHg within 30 minutes.

What is the next step in management?
A. Continue labetalol and monitor closely for rebound hypertension
B. Discontinue antihypertensive therapy immediately
C. Transition to oral nifedipine and discharge home
D. Repeat labetalol to further reduce BP to <120/80 mmHg

A

A. Continue labetalol and monitor closely for rebound hypertension
Rationale: Gradual BP reduction to the target range of 140-150/90-100 mmHg is appropriate. Antihypertensive therapy should not be discontinued abruptly, and rebound hypertension must be prevented.

54
Q

A 30-year-old G1P0 with severe preeclampsia at 35 weeks gestation is receiving oral nifedipine for BP control. BP is now 145/95 mmHg, and fetal status is reassuring.

What is the primary concern if BP is lowered excessively?
A. Worsening maternal proteinuria
B. Increased risk of preterm labor
C. Decreased uteroplacental perfusion leading to fetal hypoxia
D. Increased risk of cerebrovascular accident (CVA)

A

C. Decreased uteroplacental perfusion leading to fetal hypoxia
Rationale: Excessive BP lowering can reduce uteroplacental blood flow, causing fetal hypoxia or distress. Gradual reduction of BP to the target range minimizes this risk.

55
Q

A 30-year-old G2P1 at 34 weeks gestation presents with BP 170/115 mmHg and severe headache. Fetal heart rate tracing is reassuring. The physician plans to administer an antihypertensive agent for acute BP control.

Which of the following medications is the most appropriate first-line choice?
A. Oral nifedipine
B. Sublingual nifedipine
C. IV labetalol
D. Oral hydralazine

Follow-Up Question
What is the primary goal of using IV labetalol in this scenario?
A. Normalize BP to <120/80 mmHg
B. Gradually reduce BP to a target range of 140-150/90-100 mmHg
C. Prevent proteinuria progression
D. Minimize fetal growth restriction

A

C. IV labetalol
Rationale: IV labetalol is effective for rapid BP control in hypertensive emergencies during pregnancy. Sublingual nifedipine is contraindicated due to the risk of a sudden BP drop, which can compromise uteroplacental blood flow. Oral hydralazine is less commonly used.

B. Gradually reduce BP to a target range of 140-150/90-100 mmHg
Rationale: The goal of antihypertensive therapy in severe preeclampsia is to achieve a safe BP range of 140-150/90-100 mmHg, preventing complications such as stroke while maintaining uteroplacental perfusion.

56
Q

A 28-year-old G1P0 with severe preeclampsia at 36 weeks gestation is started on oral nifedipine for BP 165/105 mmHg.

What is the key consideration when using nifedipine in this patient?
A. Administer sublingually to achieve rapid BP control
B. Combine with IV labetalol to enhance efficacy
C. Avoid sublingual administration to prevent fetal distress
D. Discontinue if proteinuria increases

Follow-Up Question
What advantage does nifedipine have over hydralazine in this scenario?
A. Lower risk of maternal hypotension
B. Rapid onset of action
C. Reduced risk of persistent severe hypertension
D. Less frequent dosing required for maintenance

A

Answer: C. Avoid sublingual administration to prevent fetal distress
Rationale: Sublingual nifedipine causes a rapid BP drop, increasing the risk of fetal distress due to reduced uteroplacental blood flow. Oral administration ensures gradual BP reduction and maintains fetal safety.

Answer: D. Less frequent dosing required for maintenance
Rationale: Oral nifedipine is effective for maintenance therapy with less frequent dosing compared to hydralazine, making it suitable for sustained BP control in preeclampsia.

57
Q

A 35-year-old G3P2 at 30 weeks gestation presents with BP 160/110 mmHg. IV hydralazine is initiated for acute BP control.

What complication should be monitored during hydralazine therapy?
A. Severe bradycardia
B. Abrupt fetal growth restriction
C. Maternal hypotension
D. Persistent proteinuria

Follow-Up Question
If maternal hypotension occurs during hydralazine therapy, what is the most appropriate next step?
A. Discontinue hydralazine and observe without intervention
B. Administer IV fluids and titrate the dose upward
C. Switch to oral nifedipine for maintenance therapy
D. Perform emergency cesarean delivery

A

Answer: C. Maternal hypotension
Rationale: Hydralazine can cause maternal hypotension if not carefully titrated, which may reduce uteroplacental perfusion and result in fetal hypoxia.

Answer: B. Administer IV fluids and titrate the dose upward
Rationale: Volume depletion in preeclampsia may exacerbate hypotension during hydralazine therapy. Administering IV fluids and carefully titrating the dose helps maintain uteroplacental perfusion.

58
Q

A 29-year-old G1P0 at 32 weeks gestation is started on IV labetalol for severe hypertension (BP 175/115 mmHg). After 30 minutes, her BP decreases to 140/90 mmHg.

What is the next step in management?
A. Discontinue labetalol as BP is controlled
B. Transition to oral labetalol for maintenance therapy
C. Continue IV labetalol and monitor for rebound hypertension
D. Add oral nifedipine to further reduce BP

A

C. Continue IV labetalol and monitor for rebound hypertension
Rationale: Gradual reduction of BP to the target range should be maintained. Rebound hypertension is a concern, and IV labetalol should be continued with close monitoring before transitioning to oral therapy.

59
Q

A 34-year-old G2P1 presents at 35 weeks gestation with BP of 175/115 mmHg. She is started on IV labetalol.

What is the initial dosage of labetalol for this patient?
A. 5 mg IV bolus, then increase every 20 minutes
B. 10-20 mg IV bolus, then increase to 20-80 mg IV every 20-30 minutes
C. 10-20 mg IV bolus, then infusion at 2-4 mg/min
D. 1 mg/min IV infusion, titrate hourly

What is the maximum cumulative dose of IV labetalol for this patient?
A. 200 mg
B. 250 mg
C. 300 mg
D. 400 mg

A

B. 10-20 mg IV bolus, then increase to 20-80 mg IV every 20-30 minutes
Rationale: IV labetalol is initiated with a bolus of 10-20 mg and titrated every 20-30 minutes as needed. It is the preferred first-line agent for severe hypertension due to its rapid onset and safety profile.

C. 300 mg
Rationale: The maximum dose of IV labetalol is 300 mg. Beyond this, other antihypertensives such as hydralazine or nicardipine may be considered if BP remains uncontrolled.

60
Q

A 30-year-old G1P0 at 34 weeks gestation is treated with IV hydralazine for BP 165/110 mmHg.

What is the recommended initial dose of hydralazine?
A. 5 mg IV/IM, then 5-10 mg every 20-40 minutes
B. 20 mg IV bolus, then 10 mg every hour
C. 10 mg IV infusion, titrate by 1 mg/hr increments
D. 2 mg IV bolus, then repeat every 2 hours

What adverse effect should be monitored during hydralazine therapy?
A. Hyperkalemia
B. Maternal hypotension and reflex tachycardia
C. Pulmonary edema
D. Bradycardia

A

A. 5 mg IV/IM, then 5-10 mg every 20-40 minutes
Rationale: The recommended dose for hydralazine is 5 mg IV or IM initially, followed by 5-10 mg every 20-40 minutes as needed, with a maximum dose of 20 mg IV.

B. Maternal hypotension and reflex tachycardia
Rationale: Hydralazine can cause maternal hypotension, which may reduce uteroplacental blood flow, and reflex tachycardia. Gradual dose titration is necessary to minimize these risks.

61
Q

A 28-year-old G2P1 with severe preeclampsia at 32 weeks gestation is started on oral nifedipine for BP 170/110 mmHg.

What is the correct initial dosing strategy for oral nifedipine in this patient?
A. 10 mg orally every 6 hours
B. 10-20 mg orally, repeat after 30 minutes if needed
C. 10 mg sublingually, repeat every 1 hour
D. 5 mg orally, repeat every 2 hours

What is the most important precaution when using oral nifedipine for BP control?
A. Avoid doses higher than 30 mg in 24 hours
B. Administer sublingually for rapid action
C. Ensure gradual BP reduction to prevent fetal hypoxia
D. Combine with thiazide diuretics to enhance efficacy

A

B. 10-20 mg orally, repeat after 30 minutes if needed
Rationale: Oral nifedipine is administered at an initial dose of 10-20 mg, with a repeat dose after 30 minutes if BP remains elevated. Sublingual administration is avoided due to the risk of rapid BP drops and fetal distress.

C. Ensure gradual BP reduction to prevent fetal hypoxia
Rationale: Gradual BP reduction with oral nifedipine minimizes the risk of fetal hypoxia and uteroplacental insufficiency, especially in preeclampsia.

62
Q

A 32-year-old G3P2 at 31 weeks gestation is receiving IV nicardipine for BP 180/115 mmHg.

What is the recommended initial infusion rate for nicardipine?
A. 2 mg/hr, titrate every 30 minutes
B. 5 mg/hr, increase hourly by 1 mg/hr increments
C. 10 gtts/min, titrate hourly by 1 mg/hr increments
D. 1 mg/hr, titrate by 0.5 mg/hr every 15 minutes

When is IV nicardipine preferred over other antihypertensives?
A. In mild gestational hypertension
B. In refractory severe hypertension requiring rapid BP control
C. As a first-line agent for all preeclampsia cases
D. For managing proteinuria in preeclampsia

A

C. 10 gtts/min, titrate hourly by 1 mg/hr increments
Rationale: IV nicardipine is typically started at 10 gtts/min, with hourly titration by 1 mg/hr increments to a maximum dose of 10 mg/hr, ensuring controlled BP reduction.

B. In refractory severe hypertension requiring rapid BP control
Rationale: IV nicardipine is preferred in severe, refractory hypertension when rapid BP control is required, especially in critical situations where other agents have failed.

63
Q

A 35-year-old G1P0 at 33 weeks gestation develops pulmonary edema while receiving antihypertensive therapy for severe preeclampsia.

Which medication is indicated in this scenario?
A. IV labetalol
B. Oral nifedipine
C. IV furosemide
D. IV hydralazine

A

C. IV furosemide
Rationale: Furosemide is indicated for managing pulmonary edema in preeclampsia to relieve fluid overload. It is not routinely used for BP control due to potential hypokalemia and volume depletion.

64
Q

A 28-year-old G1P0 at 30 weeks gestation presents with BP 145/95 mmHg. She is asymptomatic, and fetal status is reassuring. The physician plans to initiate oral antihypertensive therapy.

Which of the following is the most appropriate initial medication?
A. Oral nifedipine
B. Oral labetalol
C. Methyldopa
D. Thiazide diuretics

What is the recommended starting dose of methyldopa for this patient?
A. 500 mg/day in divided doses
B. 250 mg once daily
C. 1,000 mg/day in divided doses
D. 2,000 mg once daily

A

C. Methyldopa
Rationale: Methyldopa is the safest and most commonly used oral antihypertensive for long-term BP control in pregnancy. It is particularly effective for mild/moderate hypertension. Labetalol oral formulation is not available in the Philippines, and thiazides are only indicated for pulmonary edema.

A. 500 mg/day in divided doses
Rationale: Methyldopa is typically started at 500 mg/day in divided doses and can be titrated to a maximum of 3 g/day depending on the patient’s response.

65
Q

A 35-year-old G2P1 with gestational hypertension at 32 weeks gestation is started on slow-release oral nifedipine.

What is the recommended dosage for nifedipine in this patient?
A. 30-120 mg/day orally using slow-release preparations
B. 10-20 mg orally every 2-6 hours
C. 5-10 mg orally every 8 hours
D. 40-80 mg/day orally using sublingual preparations

Why is slow-release nifedipine preferred over immediate-release formulations for outpatient management?
A. It has fewer side effects
B. It reduces the risk of fetal hypoxia from abrupt BP changes
C. It is more effective for severe hypertension
D. It has a higher bioavailability

A

A. 30-120 mg/day orally using slow-release preparations
Rationale: Slow-release oral nifedipine is preferred for outpatient management or long-term control of mild to moderate hypertension in pregnancy. Sublingual administration is contraindicated due to the risk of sudden BP drops.

B. It reduces the risk of fetal hypoxia from abrupt BP changes
Rationale: Slow-release nifedipine ensures gradual BP reduction, minimizing the risk of maternal hypotension and fetal hypoxia, which are associated with rapid BP changes seen in immediate-release formulations.

66
Q

A 30-year-old G3P2 with a history of preeclampsia is currently 28 weeks pregnant and has a BP of 140/90 mmHg. The physician is considering methyldopa for long-term control.

What is the maximum daily dose of methyldopa that can be prescribed for this patient?
A. 1 g/day
B. 2 g/day
C. 3 g/day
D. 4 g/day

What is the primary mechanism by which methyldopa lowers BP?
A. Inhibition of calcium channels
B. Peripheral vasodilation
C. Central alpha-adrenergic agonism
D. Blockade of beta-adrenergic receptors

A

C. 3 g/day
Rationale: Methyldopa is prescribed at a dosage of 0.5-3 g/day in divided doses. The maximum daily dose is 3 g, titrated based on the patient’s response and tolerance.

C. Central alpha-adrenergic agonism
Rationale: Methyldopa is a centrally acting alpha-adrenergic agonist that reduces sympathetic outflow, thereby lowering BP.

67
Q

A 31-year-old G1P0 at 33 weeks gestation presents with pulmonary edema and severe preeclampsia.

Which of the following oral antihypertensives is contraindicated for routine BP management in this patient?
A. Methyldopa
B. Oral nifedipine
C. Thiazide diuretics
D. Oral labetalol

What is the most appropriate initial therapy for pulmonary edema in this patient?
A. IV furosemide
B. Oral nifedipine
C. Slow-release methyldopa
D. Continuous labetalol infusion

A

C. Thiazide diuretics
Rationale: Thiazide diuretics are reserved for managing pulmonary edema in preeclampsia but are not used for routine BP control due to risks of hypokalemia and volume depletion.

A. IV furosemide
Rationale: Pulmonary edema in preeclampsia is treated with IV furosemide to manage fluid overload. Antihypertensives like methyldopa or nifedipine address BP control but do not directly resolve pulmonary edema.

68
Q

A 28-year-old G1P0 at 30 weeks gestation presents with BP 155/95 mmHg and mild proteinuria. Her previous physician prescribed enalapril for chronic hypertension.

What is the most appropriate next step in her management?
A. Continue enalapril and monitor fetal well-being
B. Switch to methyldopa for BP control
C. Initiate oral nifedipine and taper off enalapril
D. Discontinue enalapril and monitor without antihypertensive therapy

What is the fetal risk associated with continued ACE inhibitor use during pregnancy?
A. Fetal macrosomia and polyhydramnios
B. Renal dysgenesis and oligohydramnios
C. Premature labor and fetal growth acceleration
D. Cardiac defects and excessive fetal weight gain

A

B. Switch to methyldopa for BP control
Rationale: ACE inhibitors, such as enalapril, are contraindicated during pregnancy due to teratogenic effects and risks of fetal renal dysgenesis, oligohydramnios, and neonatal morbidity. Methyldopa is a safe alternative for BP control in pregnancy.

B. Renal dysgenesis and oligohydramnios
Rationale: ACE inhibitors can cause fetal renal dysgenesis, oligohydramnios, and growth restriction, along with neonatal morbidity and mortality.

69
Q

A 32-year-old G2P1 at 34 weeks gestation with preeclampsia is admitted for severe hypertension (BP 165/110 mmHg). The physician is considering antihypertensive options.

Which of the following is contraindicated for BP management in this patient?
A. Oral nifedipine
B. IV hydralazine
C. Labetalol
D. Losartan

Which of the following postpartum scenarios would justify the use of losartan?
A. Severe postpartum hypertension with renal dysfunction
B. Postpartum fluid overload without hypertension
C. Mild gestational hypertension resolving after delivery
D. Chronic hypertension requiring long-term BP control

A

D. Losartan
Rationale: Angiotensin receptor blockers (ARBs), such as losartan, are contraindicated in pregnancy due to similar risks to ACE inhibitors, including fetal renal damage and oligohydramnios.

D. Chronic hypertension requiring long-term BP control
Rationale: Losartan can be safely reintroduced postpartum for the long-term management of chronic hypertension.

70
Q

A 35-year-old G3P2 at 36 weeks gestation with severe preeclampsia develops pulmonary edema.

Which of the following medications is appropriate for this patient?
A. Furosemide
B. Enalapril
C. Hydrochlorothiazide
D. Losartan

What is the primary risk of using diuretics routinely in pregnancy?
A. Fetal hyperkalemia
B. Maternal hypoglycemia
C. Reduced uteroplacental blood flow
D. Increased risk of preterm labor

A

A. Furosemide
Rationale: Diuretics, such as furosemide, are reserved for specific indications like pulmonary edema in pregnancy. Other diuretics and drugs like enalapril and losartan are contraindicated due to fetal risks.

C. Reduced uteroplacental blood flow
Rationale: Diuretics can lead to volume depletion and reduced uteroplacental blood flow, increasing the risk of fetal hypoxia and distress.

71
Q

A 30-year-old G1P0 with chronic hypertension delivers at 38 weeks. Her BP is initially normal but rises to 160/105 mmHg on postpartum day 2.

Which medication is most appropriate for BP control in this patient?
A. Enalapril
B. Methyldopa
C. Nifedipine
D. Hydralazine

What is the typical postpartum BP pattern that necessitates close monitoring?
A. BP spikes in the first 12 hours after delivery
B. Initial BP normalization followed by increases after 24 hours
C. Gradual BP decline over the first week postpartum
D. Consistently high BP throughout the postpartum period

A

A. Enalapril
Rationale: ACE inhibitors, such as enalapril, are safe postpartum and effective for managing chronic or postpartum hypertension. Methyldopa is less effective postpartum and is typically used during pregnancy.

B. Initial BP normalization followed by increases after 24 hours
Rationale: BP often normalizes in the first 12-24 hours postpartum but then increases due to hemodynamic adjustments, requiring close monitoring and management.

72
Q

A 28-year-old G2P1 with chronic hypertension was managed on losartan prior to pregnancy. She delivers at 37 weeks and her BP is 150/95 mmHg postpartum.

What is the most appropriate management?
A. Restart losartan postpartum
B. Initiate methyldopa for BP control
C. Monitor BP without initiating therapy
D. Start nifedipine slow-release

A

A. Restart losartan postpartum
Rationale: Losartan can be safely restarted postpartum for chronic hypertension, as it is no longer contraindicated after delivery. Methyldopa and nifedipine are more commonly used during pregnancy.

73
Q

A 29-year-old G1P0 at 32 weeks gestation presents with preterm labor. The physician decides to administer antenatal corticosteroids to enhance fetal lung maturity.

What is the most appropriate regimen for this patient?
A. Betamethasone 12 mg IM every 24 hours for 2 doses
B. Betamethasone 6 mg IM every 12 hours for 4 doses
C. Dexamethasone 12 mg IM every 24 hours for 2 doses
D. Dexamethasone 6 mg IM every 12 hours for 4 doses

Follow-Up Question
What is the ideal timing for delivery after completing the corticosteroid regimen?
A. Immediately after the last dose
B. 12-24 hours after the last dose
C. 24-48 hours after the last dose
D. 7 days after the last dose

A

A. Betamethasone 12 mg IM every 24 hours for 2 doses
Rationale: Betamethasone at 12 mg IM every 24 hours for 2 doses is a standard regimen for enhancing fetal lung maturity. Dexamethasone is also an option, administered as 6 mg IM every 12 hours for 4 doses.

C. 24-48 hours after the last dose
Rationale: Delivery is ideally timed 24-48 hours after completing corticosteroid administration to maximize its effects on fetal lung maturity and reduce neonatal complications.

74
Q

A 30-year-old G2P1 at 34+2 weeks gestation with preeclampsia has not received antenatal corticosteroids. The physician is considering whether to administer them before planned delivery.

What is the most appropriate course of action?
A. Administer corticosteroids and delay delivery for 7 days
B. Administer corticosteroids and plan delivery in 24-48 hours
C. Do not administer corticosteroids after 34 weeks
D. Administer corticosteroids only if delivery is within 12 hours

What is the primary benefit of corticosteroids in late preterm gestations (>34 to <36 weeks)?
A. Promoting fetal lung maturity
B. Reducing the risk of neonatal sepsis
C. Decreasing the incidence of RDS, cerebrovascular hemorrhage, and NEC
D. Preventing preterm labor

A

B. Administer corticosteroids and plan delivery in 24-48 hours
Rationale: Antenatal corticosteroids can still be beneficial between 34 and 36 weeks, reducing neonatal complications like cerebrovascular hemorrhage and necrotizing enterocolitis. Delivery should occur 24-48 hours after completing the regimen if possible.

C. Decreasing the incidence of RDS, cerebrovascular hemorrhage, and NEC
Rationale: While the primary benefit of corticosteroids at <34 weeks is lung maturity, in late preterm gestations, they help reduce complications like RDS, cerebrovascular hemorrhage, and NEC.

75
Q

A 28-year-old G3P2 at 26 weeks gestation is at risk for preterm delivery within the next week. She receives her first dose of betamethasone.

What is the next step in her management regarding corticosteroids?
A. Administer the second dose 12 hours after the first
B. Administer the second dose 24 hours after the first
C. Complete the regimen only if delivery is imminent within 12 hours
D. Repeat the entire regimen weekly until delivery

A

B. Administer the second dose 24 hours after the first
Rationale: Betamethasone is given as two doses 24 hours apart to optimize fetal lung maturity. Weekly repeat doses are not routinely recommended.

76
Q

A 35-year-old G2P1 at 30 weeks gestation presents with premature rupture of membranes (PROM). The physician plans to administer dexamethasone.

What is the correct regimen for dexamethasone in this scenario?
A. 6 mg IM every 12 hours for 2 doses
B. 6 mg IM every 12 hours for 4 doses
C. 12 mg IM every 24 hours for 2 doses
D. 12 mg IM every 12 hours for 4 doses

A

B. 6 mg IM every 12 hours for 4 doses
Rationale: Dexamethasone is administered as 6 mg IM every 12 hours for 4 doses to enhance fetal lung maturity and reduce neonatal complications.

77
Q

A 32-year-old G1P0 with preterm labor at 33 weeks gestation is started on antenatal corticosteroids. She delivers 6 hours after completing the second dose of betamethasone.

Which neonatal complications are least likely to occur due to corticosteroid administration?
A. Respiratory distress syndrome (RDS)
B. Necrotizing enterocolitis (NEC)
C. Neonatal sepsis
D. Cerebrovascular hemorrhage

A

A. Respiratory distress syndrome (RDS)
Rationale: Antenatal corticosteroids significantly reduce the risk of RDS, as well as other complications like NEC and cerebrovascular hemorrhage. Neonatal sepsis is not directly prevented by corticosteroids.

78
Q

A 32-year-old G1P0 at 36 weeks gestation presents with BP 165/110 mmHg, severe headache, and blurred vision. The physician diagnoses preeclampsia with severe features.

What is the primary indication for administering magnesium sulfate in this patient?
A. To control severe hypertension
B. To prevent progression to eclampsia
C. To treat proteinuria
D. To enhance fetal lung maturity

What is the appropriate loading dose of magnesium sulfate for this patient?
A. 4 g IV over 5-10 minutes
B. 10 g IM (5 g per buttock)
C. 4 g IV over 30 minutes
D. 6 g IV over 5-10 minutes

A

B. To prevent progression to eclampsia
Rationale: Magnesium sulfate is the drug of choice for preventing seizures in patients with preeclampsia with severe features, reducing the risk of progression to eclampsia.

A. 4 g IV over 5-10 minutes
Rationale: The standard loading dose for magnesium sulfate is 4 g IV administered slowly over 5-10 minutes. This may be followed by maintenance doses via IV infusion or IM injections.

79
Q

A 28-year-old G2P1 with preeclampsia without severe features is being managed expectantly at 34 weeks gestation. The physician is considering magnesium sulfate.

Should magnesium sulfate be administered in this case?
A. Yes, to prevent seizures in all cases of preeclampsia
B. No, it is not indicated for preeclampsia without severe features
C. Yes, as it reduces the risk of hypertension-related complications
D. No, unless the patient progresses to eclampsia

What alternative therapy should be prioritized for this patient?
A. Antihypertensive therapy to control BP
B. Calcium gluconate to prevent neuromuscular toxicity
C. IV diazepam for seizure prophylaxis
D. Immediate delivery to prevent progression

A

B. No, it is not indicated for preeclampsia without severe features
Rationale: Magnesium sulfate is not routinely administered in preeclampsia without severe features due to the risks of toxicity and side effects. It is reserved for cases with severe features or eclampsia.

A. Antihypertensive therapy to control BP
Rationale: In preeclampsia without severe features, controlling BP with antihypertensives is the priority to prevent complications. Magnesium sulfate is not indicated unless severe features develop.

80
Q

A 34-year-old G3P2 with eclampsia is admitted after experiencing a tonic-clonic seizure. The physician plans to administer magnesium sulfate.

What is the appropriate loading dose and route for magnesium sulfate in this scenario?
A. 4 g IV over 5-10 minutes
B. 10 g IM (5 g per buttock)
C. 4 g IV over 30 minutes, followed by 2 g/hour infusion
D. 10 g IV bolus

What is the next step if the patient develops signs of magnesium sulfate toxicity (e.g., loss of reflexes, respiratory depression)?
A. Discontinue magnesium sulfate and observe
B. Administer calcium gluconate 10 mL of 10% solution IV
C. Reduce the magnesium sulfate infusion rate
D. Switch to diazepam for seizure prophylaxis

A

A. 4 g IV over 5-10 minutes
Rationale: The standard loading dose for treating eclampsia is 4 g IV over 5-10 minutes. This can be followed by maintenance doses via IV infusion or IM injections.

B. Administer calcium gluconate 10 mL of 10% solution IV
Rationale: Calcium gluconate is the antidote for magnesium sulfate toxicity and should be administered immediately if signs of toxicity are observed.

81
Q

A 29-year-old G1P0 at 30 weeks gestation is being treated with a continuous IV infusion of magnesium sulfate for preeclampsia with severe features.

What maintenance dose should be used in this patient?
A. 2 g/hour infusion after a 4 g IV loading dose
B. 1 g/hour infusion after a 4 g IV loading dose
C. 5 g IM every 4 hours after a 10 g loading dose
D. 20 g IV bolus followed by 1 g/hour infusion

A

A. 2 g/hour infusion after a 4 g IV loading dose
Rationale: The standard maintenance dose for magnesium sulfate is 2 g/hour after a 4 g IV loading dose.

82
Q

A 33-year-old G2P1 at 37 weeks gestation is receiving magnesium sulfate for severe preeclampsia. She develops respiratory depression with a respiratory rate of 10 breaths/min and absent deep tendon reflexes.

What is the immediate course of action?
A. Discontinue magnesium sulfate and monitor the patient
B. Administer calcium gluconate 10 mL of 10% solution IV
C. Reduce the magnesium sulfate infusion rate to 1 g/hour
D. Intubate the patient and continue magnesium sulfate

A

B. Administer calcium gluconate 10 mL of 10% solution IV
Rationale: Calcium gluconate is the antidote for magnesium sulfate toxicity and should be administered immediately to reverse neuromuscular blockade and respiratory depression.

83
Q

A 30-year-old G2P1 at 34 weeks gestation presents to the emergency room with a tonic-clonic seizure. The physician plans to administer an anticonvulsant to control the seizure immediately.

Which medication is the most appropriate for immediate seizure control?
A. Magnesium sulfate
B. Diazepam
C. Phenytoin
D. Nifedipine

What is the recommended loading dose of diazepam for this patient?
A. 4 g IV over 5 minutes
B. 10 mg IV over 2 minutes
C. 1 g IV infused slowly over 20 minutes
D. 6 mg IV every 12 hours

A

B. Diazepam
Rationale: Diazepam is the drug of choice for acute convulsions due to its rapid onset of action (1 minute), which is critical for immediate seizure control. Magnesium sulfate, while effective in preventing recurrent seizures, has a slower onset and is not suitable for acute seizure management.

B. 10 mg IV over 2 minutes
Rationale: The loading dose of diazepam for acute seizure control is 10 mg IV administered over 2 minutes to ensure rapid seizure termination.

84
Q

A 35-year-old G3P2 with eclampsia experiences recurrent seizures despite initial diazepam administration. The physician decides to add another anticonvulsant.

Which medication is the next best choice for managing the recurrent seizures?
A. Magnesium sulfate
B. Phenytoin
C. Labetalol
D. Hydralazine

What is the appropriate loading dose of phenytoin for this patient?
A. 10 mg IV over 2 minutes
B. 1 g IV infused slowly over 20 minutes
C. 40 mg diluted in 500 mL Normal Saline over 24 hours
D. 6 mg IM every 12 hours

A

B. Phenytoin
Rationale: Phenytoin is an alternative to diazepam for managing acute seizures and is effective when seizures recur. It works by blocking sodium channels in neurons to reduce excitability.

B. 1 g IV infused slowly over 20 minutes
Rationale: The loading dose of phenytoin is 1 g IV infused over 20 minutes to prevent further seizures. Slow infusion is critical to avoid complications such as hypotension.

85
Q

A 28-year-old G1P0 at 36 weeks gestation presents with eclampsia. The seizure is controlled with diazepam. The physician plans to prevent seizure recurrence.

Which medication is the most appropriate for preventing seizure recurrence in this patient?
A. Diazepam infusion
B. Magnesium sulfate
C. Phenytoin infusion
D. Labetalol

What is the recommended maintenance dose of magnesium sulfate in this patient?
A. 5 g IM every 4 hours
B. 2 g/hour IV infusion after a 4 g IV loading dose
C. 10 mg IV every 12 hours
D. 1 g IV every 6 hours

A

B. Magnesium sulfate
Rationale: Magnesium sulfate is the drug of choice for preventing seizure recurrence in patients with eclampsia. It works peripherally to block calcium channels and reduce neuromuscular excitability.

B. 2 g/hour IV infusion after a 4 g IV loading dose
Rationale: After a 4 g IV loading dose, magnesium sulfate is maintained at 2 g/hour via IV infusion to ensure therapeutic levels and prevent recurrent seizures.

86
Q

A 32-year-old G2P1 with eclampsia develops hypotension and respiratory depression during phenytoin infusion.

What is the most appropriate next step?
A. Administer calcium gluconate IV
B. Stop the infusion and provide supportive care
C. Switch to diazepam for seizure control
D. Increase the infusion rate to complete the dose quickly

A

B. Stop the infusion and provide supportive care
Rationale: Hypotension and respiratory depression are potential side effects of phenytoin infusion. The infusion should be stopped, and supportive measures such as oxygen therapy should be initiated.

87
Q

A 30-year-old G3P2 at 33 weeks gestation presents with preeclampsia with severe features and is receiving magnesium sulfate prophylaxis. She develops a seizure during administration.

What is the best course of action for immediate seizure control?
A. Increase the magnesium sulfate infusion rate
B. Administer diazepam 10 mg IV over 2 minutes
C. Administer a bolus dose of phenytoin
D. Intubate the patient and continue magnesium sulfate

A

B. Administer diazepam 10 mg IV over 2 minutes
Rationale: During an active seizure, diazepam is the drug of choice for rapid seizure control. Magnesium sulfate acts peripherally and is not effective for managing acute seizures.

88
Q

A 32-year-old G1P0 at 37 weeks gestation presents with stable preeclampsia without severe features. Both maternal and fetal conditions are reassuring.

What is the recommended timing of delivery for this patient?
A. Immediate delivery
B. Expectant management until 39 weeks gestation
C. Delivery at or beyond 37 weeks gestation
D. Expectant management until 40 weeks gestation

What delivery method should be prioritized for this patient?
A. Cesarean section
B. Induction of labor with prostaglandins and/or oxytocin
C. Elective cesarean section at 39 weeks
D. Emergency cesarean section

A

C. Delivery at or beyond 37 weeks gestation
Rationale: For stable preeclampsia without severe features, delivery is recommended at or beyond 37 weeks gestation to minimize maternal and fetal risks.

B. Induction of labor with prostaglandins and/or oxytocin
Rationale: Vaginal delivery is often attempted unless contraindicated. Induction of labor is preferred unless there are obstetric complications requiring cesarean delivery.

89
Q

A 29-year-old G2P1 at 33 weeks gestation is diagnosed with HELLP syndrome. Maternal BP is 170/115 mmHg, and platelet count is 85,000/mm³. The patient has received corticosteroids for fetal lung maturity.

What is the next best step in management?
A. Continue expectant management for 2 more weeks
B. Perform immediate delivery after maternal stabilization
C. Delay delivery until fetal lung maturity is confirmed
D. Administer magnesium sulfate and repeat corticosteroids

What additional management step is critical before delivery in this patient?
A. Administer antihypertensives and magnesium sulfate
B. Perform an emergency cesarean section without stabilization
C. Observe for 24 hours and reassess maternal status
D. Perform a non-stress test before deciding on delivery

A

B. Perform immediate delivery after maternal stabilization
Rationale: HELLP syndrome is an indication for immediate delivery at any gestational age once maternal stabilization is achieved, as delaying delivery increases the risk of severe maternal complications.

A. Administer antihypertensives and magnesium sulfate
Rationale: Stabilization with antihypertensives and magnesium sulfate is critical to prevent complications such as stroke or eclampsia before delivery.

90
Q

A 30-year-old G3P2 at 35 weeks gestation with preeclampsia with severe features is being monitored. Maternal BP is 165/110 mmHg despite treatment, and fetal Doppler studies show abnormal umbilical artery flow.

What is the most appropriate next step in management?
A. Immediate delivery regardless of maternal or fetal status
B. Expectant management with daily fetal monitoring
C. Administer corticosteroids and delay delivery for 48 hours
D. Initiate labor induction with close monitoring

What delivery method should be used if the fetal heart rate tracing is non-reassuring?
A. Induction of labor with oxytocin
B. Immediate cesarean section
C. Vaginal delivery under close fetal monitoring
D. Administration of magnesium sulfate and observe

A

A. Immediate delivery regardless of maternal or fetal status
Rationale: In preeclampsia with severe features at ≥34 weeks, delivery is expedited to prevent worsening maternal and fetal complications, regardless of maternal or fetal stability.

B. Immediate cesarean section
Rationale: Non-reassuring fetal heart rate tracings indicate fetal compromise, requiring immediate delivery via cesarean section if vaginal delivery poses additional risks.

91
Q

A 28-year-old G1P0 at 32 weeks gestation is diagnosed with preeclampsia with severe features. Both maternal BP and fetal surveillance are stable.

What is the next best step in management?
A. Immediate delivery
B. Expectant management with close monitoring
C. Delay delivery for 2-3 weeks without intervention
D. Emergency cesarean section

A

B. Expectant management with close monitoring
Rationale: In preeclampsia with severe features at <34 weeks, expectant management can be considered if maternal and fetal conditions are stable, along with close monitoring and corticosteroid administration.

92
Q

A 35-year-old G2P1 at 33 weeks gestation presents with premature rupture of membranes (PROM) and preeclampsia without severe features. Maternal and fetal conditions are stable.

What is the recommended timing of delivery?
A. Immediate delivery to prevent infection
B. Expectant management with daily NSTs and biophysical profiles
C. Induction of labor after corticosteroid administration
D. Delay delivery until 36 weeks gestation

A

C. Induction of labor after corticosteroid administration
Rationale: PROM at 33 weeks in preeclampsia requires induction of labor after corticosteroid administration to reduce neonatal complications while minimizing infection risk.

93
Q

A 29-year-old G2P1 at 34 weeks gestation with preeclampsia presents with persistent severe headache, visual disturbances, and a BP of 175/115 mmHg despite being on two antihypertensive agents.

What is the most appropriate next step?
A. Continue antihypertensive therapy and monitor closely
B. Administer magnesium sulfate and delay delivery for 48 hours
C. Immediate delivery
D. Perform a repeat non-stress test (NST)

What additional maternal condition in this patient would strengthen the indication for immediate delivery?
A. Platelet count <150,000/mm³
B. Epigastric pain with AST >2x normal
C. Amniotic fluid index of 8 cm
D. BP controlled at 150/90 mmHg with one antihypertensive

A

C. Immediate delivery
Rationale: Uncontrolled severe hypertension and persistent neurological symptoms (e.g., headache, visual changes) are maternal indications for immediate delivery to prevent further complications such as stroke or eclampsia.

B. Epigastric pain with AST >2x normal
Rationale: Epigastric pain with elevated liver enzymes is indicative of hepatic dysfunction or impending HELLP syndrome, necessitating immediate delivery.

94
Q

A 34-year-old G3P2 at 32 weeks gestation with preeclampsia is being managed expectantly. Fetal Doppler studies show absent end-diastolic flow (AEDF) in the umbilical artery, and the biophysical profile (BPP) score is 2.

What is the most appropriate management for this patient?
A. Continue expectant management with daily fetal monitoring
B. Administer corticosteroids and delay delivery for 48 hours
C. Immediate delivery due to fetal indications
D. Repeat BPP in 6 hours and reassess

Which additional fetal finding would confirm the need for immediate delivery?
A. Persistent severe oligohydramnios
B. FGR with estimated weight at the 20th percentile
C. Reactive NST with occasional decelerations
D. Biophysical profile score improving to 6

A

C. Immediate delivery due to fetal indications
Rationale: Absent end-diastolic flow (AEDF) and a persistent BPP <4 are strong fetal indications for immediate delivery to prevent worsening hypoxia and fetal death.

A. Persistent severe oligohydramnios
Rationale: Severe oligohydramnios and abnormal Doppler findings indicate placental insufficiency and fetal distress, necessitating immediate delivery.

95
Q

A 30-year-old G1P0 at 35 weeks gestation presents with preeclampsia with severe features. Laboratory results show a platelet count of 85,000/mm³, AST 120 U/L, and serum creatinine 1.8 mg/dL.

What is the most appropriate management?
A. Continue expectant management and monitor platelets daily
B. Administer magnesium sulfate and plan delivery within 24 hours
C. Delay delivery until fetal lung maturity is confirmed
D. Initiate labor induction after platelet transfusion

What additional fetal condition would support the decision for immediate delivery?
A. Estimated fetal weight <5th percentile
B. Normal umbilical artery Doppler findings
C. Amniotic fluid index of 12 cm
D. FHR with mild variable decelerations

A

B. Administer magnesium sulfate and plan delivery within 24 hours
Rationale: Thrombocytopenia (<100,000/mm³), elevated liver enzymes, and renal impairment are maternal indications for expedited delivery in preeclampsia with severe features.

A. Estimated fetal weight <5th percentile
Rationale: Severe fetal growth restriction (<5th percentile) is indicative of placental insufficiency, warranting immediate delivery.

96
Q

A 32-year-old G2P1 at 31 weeks gestation is diagnosed with preeclampsia with severe features. She reports epigastric pain and has an AST of 150 U/L. The fetal NST is non-reactive, and Doppler studies show reversed end-diastolic flow (RED).

What is the most appropriate next step in management?
A. Continue expectant management with corticosteroid administration
B. Emergency cesarean delivery
C. Induce labor with prostaglandins
D. Perform a repeat Doppler study in 24 hours

What other fetal condition would necessitate immediate delivery in this case?
A. BPP score improving to 6
B. Severe oligohydramnios with amniotic fluid index <2 cm
C. Persistent mild variable decelerations
D. Estimated fetal weight at the 10th percentile

A

B. Emergency cesarean delivery
Rationale: Reversed end-diastolic flow (RED) and non-reassuring fetal heart tracing in the presence of maternal hepatic dysfunction are strong indications for emergency cesarean delivery to prevent maternal and fetal morbidity.

B. Severe oligohydramnios with amniotic fluid index <2 cm
Rationale: Severe oligohydramnios is a sign of placental insufficiency, and when combined with reversed end-diastolic flow or non-reassuring fetal status, it warrants immediate delivery.

97
Q

A 30-year-old G1P0 at 33 weeks gestation with preeclampsia presents with stable maternal and fetal conditions. The fetus is in cephalic presentation, and cervical ripening is favorable.

What is the most appropriate mode of delivery?
A. Immediate cesarean section
B. Vaginal delivery with induction of labor
C. Elective cesarean section at 34 weeks
D. Expectant management until 37 weeks

What agent is most appropriate to induce labor in this patient?
A. Dinoprostone for cervical ripening
B. Oxytocin only
C. Magnesium sulfate to augment labor
D. Misoprostol for uterine stimulation

A

B. Vaginal delivery with induction of labor
Rationale: Vaginal delivery is the preferred mode of delivery in stable preeclamptic patients, especially when the fetus is in cephalic presentation and maternal and fetal conditions are stable. Induction of labor with prostaglandins and/or oxytocin can be used to facilitate vaginal delivery.

A. Dinoprostone for cervical ripening
Rationale: Dinoprostone is a prostaglandin used to ripen the cervix and promote labor in preeclamptic women. Oxytocin can be added after cervical ripening to strengthen contractions.

98
Q

A 32-year-old G2P1 at 35 weeks gestation with preeclampsia is undergoing induction of labor. After 12 hours of oxytocin administration, there is no cervical dilation, and the fetal heart rate shows late decelerations.

What is the most appropriate next step?
A. Continue oxytocin for another 12 hours
B. Perform an emergency cesarean section
C. Administer magnesium sulfate to prevent seizures
D. Attempt manual rotation of the fetal head

A

B. Perform an emergency cesarean section
Rationale: Failed induction combined with non-reassuring fetal heart rate patterns (late decelerations) is an indication for cesarean delivery to ensure maternal and fetal safety.

99
Q

A 28-year-old G1P0 at 36 weeks gestation with preeclampsia with severe features is being considered for delivery. The fetus is in breech presentation, and maternal BP is well-controlled.

What is the most appropriate mode of delivery?
A. Vaginal delivery with induction of labor
B. Cesarean section due to breech presentation
C. Expectant management until 37 weeks
D. External cephalic version followed by vaginal delivery

B. Cesarean section due to breech presentation
Rationale: Non-cephalic presentations, such as breech, are indications for cesarean section to reduce maternal and fetal risks. Vaginal delivery is not recommended in this case.

A

B. Cesarean section due to breech presentation
Rationale: Non-cephalic presentations, such as breech, are indications for cesarean section to reduce maternal and fetal risks. Vaginal delivery is not recommended in this case.

B. Administer magnesium sulfate for seizure prophylaxis
Rationale: Magnesium sulfate is indicated for seizure prophylaxis in preeclampsia with severe features, particularly before cesarean delivery.

100
Q

A 34-year-old G2P1 at 32 weeks gestation with preeclampsia and fetal growth restriction is undergoing induction of labor. Fetal Doppler studies show absent end-diastolic flow (AEDF).

What is the most appropriate mode of delivery?
A. Vaginal delivery if labor progresses within 6 hours
B. Cesarean section due to abnormal Doppler findings
C. Continue labor induction with oxytocin until cervical ripening is complete
D. Monitor with serial Doppler studies and delay delivery

A

B. Cesarean section due to abnormal Doppler findings
Rationale: Absent end-diastolic flow (AEDF) in fetal Doppler studies indicates significant placental insufficiency, necessitating cesarean delivery to reduce the risk of fetal hypoxia.

101
Q

A 29-year-old G3P2 at 34 weeks gestation presents with severe preeclampsia. Fetal heart rate monitoring shows persistent late decelerations. Cervix is unfavorable for induction.

What is the most appropriate mode of delivery?
A. Vaginal delivery with continued monitoring
B. Induction of labor with prostaglandins
C. Cesarean section due to fetal distress
D. Expectant management until cervix becomes favorable

A

C. Cesarean section due to fetal distress
Rationale: Persistent late decelerations suggest fetal distress, making cesarean section the safest option to prevent further fetal compromise.

102
Q

A 30-year-old G1P0 with preeclampsia is in active labor at 38 weeks gestation. Her hematocrit is elevated, and she is receiving oxytocin for labor induction.

Why is active management of the third stage of labor critical for this patient?
A. To reduce the risk of postpartum hypertension
B. To prevent hemoconcentration-related hypovolemia
C. To minimize postpartum hemorrhage (PPH) due to reduced tolerance for blood loss
D. To ensure uterine contractions remain effective

What is the preferred medication for preventing postpartum hemorrhage in this patient?
A. Misoprostol
B. Carboprost
C. Oxytocin (e.g., Duratocin)
D. Ergonovine

A

C. To minimize postpartum hemorrhage (PPH) due to reduced tolerance for blood loss
Rationale: Preeclamptic women have reduced tolerance for blood loss due to hemoconcentration and lack of normal pregnancy-induced hypervolemia. Active management of the third stage of labor, including oxytocin administration, is critical to prevent postpartum hemorrhage.

C. Oxytocin (e.g., Duratocin)
Rationale: Oxytocin is the first-line agent for stimulating uterine contraction and minimizing postpartum hemorrhage in preeclamptic patients.

103
Q

A 32-year-old G2P1 at 35 weeks gestation with preeclampsia is undergoing induction of labor. After 12 hours of oxytocin administration, there is no cervical dilation, and the fetal heart rate shows late decelerations.

What is the most appropriate next step?
A. Continue oxytocin for another 12 hours
B. Perform an emergency cesarean section
C. Administer magnesium sulfate to prevent seizures
D. Attempt manual rotation of the fetal head

A

B. Perform an emergency cesarean section
Rationale: Failed induction combined with non-reassuring fetal heart rate patterns (late decelerations) is an indication for cesarean delivery to ensure maternal and fetal safety.

104
Q

A 28-year-old G1P0 at 36 weeks gestation with preeclampsia with severe features is being considered for delivery. The fetus is in breech presentation, and maternal BP is well-controlled.

What is the most appropriate mode of delivery?
A. Vaginal delivery with induction of labor
B. Cesarean section due to breech presentation
C. Expectant management until 37 weeks
D. External cephalic version followed by vaginal delivery

What additional preparation should be considered before cesarean delivery in this patient?
A. Administer prostaglandins for cervical ripening
B. Administer magnesium sulfate for seizure prophylaxis
C. Delay surgery for 48 hours and administer corticosteroids
D. Attempt external cephalic version immediately before delivery

A

B. Cesarean section due to breech presentation
Rationale: Non-cephalic presentations, such as breech, are indications for cesarean section to reduce maternal and fetal risks. Vaginal delivery is not recommended in this case.

B. Administer magnesium sulfate for seizure prophylaxis
Rationale: Magnesium sulfate is indicated for seizure prophylaxis in preeclampsia with severe features, particularly before cesarean delivery.

105
Q

A 34-year-old G2P1 at 32 weeks gestation with preeclampsia and fetal growth restriction is undergoing induction of labor. Fetal Doppler studies show absent end-diastolic flow (AEDF).

What is the most appropriate mode of delivery?
A. Vaginal delivery if labor progresses within 6 hours
B. Cesarean section due to abnormal Doppler findings
C. Continue labor induction with oxytocin until cervical ripening is complete
D. Monitor with serial Doppler studies and delay delivery

A

B. Cesarean section due to abnormal Doppler findings
Rationale: Absent end-diastolic flow (AEDF) in fetal Doppler studies indicates significant placental insufficiency, necessitating cesarean delivery to reduce the risk of fetal hypoxia.

106
Q

A 29-year-old G3P2 at 34 weeks gestation presents with severe preeclampsia. Fetal heart rate monitoring shows persistent late decelerations. Cervix is unfavorable for induction.

What is the most appropriate mode of delivery?
A. Vaginal delivery with continued monitoring
B. Induction of labor with prostaglandins
C. Cesarean section due to fetal distress
D. Expectant management until cervix becomes favorable

A

C. Cesarean section due to fetal distress
Rationale: Persistent late decelerations suggest fetal distress, making cesarean section the safest option to prevent further fetal compromise.

107
Q

A 32-year-old G2P1 with preeclampsia delivered via cesarean section. Her BP is 145/95 mmHg on postpartum day 1, and she is receiving magnesium sulfate.

What is the most appropriate next step in her postpartum management?
A. Continue magnesium sulfate for 24 hours
B. Start methyldopa for BP control
C. Administer furosemide to manage BP
D. Discontinue magnesium sulfate immediately

At what BP threshold should antihypertensive therapy be initiated in this patient postpartum?
A. ≥140/90 mmHg
B. ≥150/100 mmHg
C. ≥160/110 mmHg
D. ≥135/85 mmHg

A

A. Continue magnesium sulfate for 24 hours
Rationale: Magnesium sulfate should be continued for 24 hours postpartum to prevent eclampsia, as the postpartum period is the maximum risk period for convulsions in preeclamptic women.

B. ≥150/100 mmHg
Rationale: Antihypertensive therapy is indicated when BP exceeds 150/100 mmHg in the postpartum period to reduce the risk of complications such as stroke.

108
Q

A 28-year-old G1P0 with preeclampsia delivered vaginally. Her BP is 160/105 mmHg on postpartum day 2. The physician decides to initiate antihypertensive therapy.

What is the most appropriate first-line antihypertensive in this case?
A. Methyldopa
B. Nifedipine
C. Furosemide
D. Hydralazine

Why is methyldopa not recommended in the postpartum period?
A. It causes renal impairment
B. It is less effective than nifedipine postpartum
C. It has a side effect of depression, which can worsen postpartum mood changes
D. It increases the risk of pulmonary edema

A

B. Nifedipine
Rationale: Nifedipine is the most commonly used antihypertensive in the postpartum period for BP control. Methyldopa is avoided postpartum due to its side effect of depression, which can exacerbate postpartum mood changes.

C. It has a side effect of depression, which can worsen postpartum mood changes
Rationale: Methyldopa is avoided postpartum due to its association with depression, which can exacerbate postpartum mood disorders.

109
Q

A 30-year-old G2P1 with severe preeclampsia is in early labor and requests pain relief. Her BP is 165/110 mmHg, and she has normal platelet levels and no neurological symptoms.

What is the preferred method for pain relief in this patient?
A. Intravenous opioids
B. Epidural anesthesia
C. General anesthesia with sedation
D. No anesthesia due to severe preeclampsia

Why should general anesthesia be avoided in this patient?
A. It increases the risk of fetal distress
B. It causes hypotension and respiratory depression
C. It increases the risk of hypertensive crises during intubation
D. It leads to prolonged labor

A

B. Epidural anesthesia
Rationale: Epidural anesthesia is the preferred method for pain relief in laboring preeclamptic women, as it reduces pain and stress, thereby controlling maternal hypertension and preventing complications.

C. It increases the risk of hypertensive crises during intubation
Rationale: General anesthesia is associated with a higher risk of hypertensive crises during intubation due to stress-induced catecholamine release, increasing the risk of intracranial hemorrhage and other complications.

110
Q

A 35-year-old G3P2 delivered via cesarean section for severe preeclampsia. She reports moderate pain on postpartum day 1 and requests pain relief.

What is the most appropriate pain management option?
A. Ibuprofen
B. Acetaminophen
C. Opioids
D. Aspirin

Why are NSAIDs generally avoided in postpartum preeclamptic patients?
A. They increase the risk of postpartum depression
B. They impair renal function and may exacerbate hypertension
C. They have less effective analgesic properties in preeclampsia
D. They interact with magnesium sulfate

A

C. Opioids
Rationale: Opioids are preferred for postpartum pain relief in preeclamptic patients to avoid potential complications from NSAIDs, such as impaired renal function or increased BP.

B. They impair renal function and may exacerbate hypertension
Rationale: NSAIDs are avoided in postpartum preeclamptic patients due to their potential to impair renal function and exacerbate hypertension, which are already concerns in this population.